Difference between revisions of "Euler's Totient Theorem Problem 2 Solution"

(Solution)
(Solution)
 
(2 intermediate revisions by 2 users not shown)
Line 3: Line 3:
  
 
==Solution==
 
==Solution==
Finding the last two digits is equivalent to finding <math>3^{3^{3^{3}}}\pmod{100}</math>. We can start by expanding the uppermost exponent: <math>3^{3^{27}}</math>. Then, since <math>\phi(100)=40</math>, the exponent is equal to <math> 3^{27}\pmod{40} </math>. We see that <math> 3^4=81\equiv1\pmod{40} </math>, so it simplifies to <math>3^3=27\pmod{40}</math>.  
+
Finding the last two digits is equivalent to finding <math>3^{3^{3^{3}}}\pmod{100}</math>. We can start by expanding the uppermost exponent: <math>3^{3^{27}}</math>. Then, since <math>\phi(100)=40</math>, the exponent is equal to <math> 3^{27}\pmod{40} </math>. We see that <math> 3^4=81\equiv1\pmod{40} </math>, so it simplifies to <math>3^3={27}\pmod{40}</math>.  
  
We now just need to find the last two digits of <math>3^27</math>. Using the [[Chinese Remainder Theorem]], we find that the last two digits are <math>3\pmod{4}</math> and <math>12\pmod{25}</math>. We guess and check to get <math>\boxed{87}</math>.
+
We now just need to find the last two digits of <math>3^{27}</math>. Using the [[Chinese Remainder Theorem]], we find that the last two digits are <math>3\pmod{4}</math> and <math>12\pmod{25}</math>. We guess and check to get <math>\boxed{87}</math>.
 
~BorealBear
 
~BorealBear
 +
 +
Link back to [[Euler's Totient Theorem]].

Latest revision as of 22:14, 15 August 2023

Problem

(BorealBear) Find the last two digits of $3^{3^{3^{3}}}$.

Solution

Finding the last two digits is equivalent to finding $3^{3^{3^{3}}}\pmod{100}$. We can start by expanding the uppermost exponent: $3^{3^{27}}$. Then, since $\phi(100)=40$, the exponent is equal to $3^{27}\pmod{40}$. We see that $3^4=81\equiv1\pmod{40}$, so it simplifies to $3^3={27}\pmod{40}$.

We now just need to find the last two digits of $3^{27}$. Using the Chinese Remainder Theorem, we find that the last two digits are $3\pmod{4}$ and $12\pmod{25}$. We guess and check to get $\boxed{87}$. ~BorealBear

Link back to Euler's Totient Theorem.